Added topic tags to some Giancoli-6 problems + minor typos
[course.git] / latex / problems / Giancoli_6 / problem19.31.tex
1 \begin{problem*}{19.31} % resistor networks
2 Calculate the currents in each resistor of Fig.~19-49.
3 \end{problem*}
4
5 \begin{nosolution}
6 \begin{center}
7 \begin{asy}
8 import Circ;
9 real u = 3cm;
10 TwoTerminal Bc = source((0,0), DC, 90, "", "$3.0\U{V}$");
11 TwoTerminal Rcb = resistor(Bc.beg, normal, -90, "$10\U{\Ohm}$", "");
12 TwoTerminal Rca = resistor(Bc.end, normal, 180, "", "$2\U{\Ohm}$");
13 pair Jtop = Rca.end, Jbot = (Jtop.x,Rcb.end.y);
14 TwoTerminal Rb = resistor(Jtop, normal, -90, "$6\U{\Ohm}$", "");
15 TwoTerminal Ba = source(Jtop, DC, 180, "", "$6.0\U{V}$");
16 TwoTerminal Rab = resistor(Jbot, normal, 180, "$8\U{\Ohm}$", "");
17 TwoTerminal Raa = resistor(Rab.end, normal, 90, "$12\U{\Ohm}$", "");
18 wire(Ba.end, Raa.end, rlsq);
19 wire(Rab.beg, Jbot, nsq);
20 wire(Jbot, Rb.end, nsq);
21 wire(Jbot, Rcb.end, rlsq);
22 \end{asy}
23 \end{center}
24 \end{nosolution}
25
26 \begin{solution}
27 \begin{center}
28 \begin{asy}
29 import Circ;
30 TwoTerminal Bc = source((0,0), DC, 90, "", "$3.0\U{V}$");
31 TwoTerminal Rcb = resistor(Bc.beg, normal, -90, "$10\U{\Ohm}$", "");
32 TwoTerminal Rca = resistor(Bc.end, normal, 180, "", "$2\U{\Ohm}$");
33 pair Jtop = Rca.end, Jbot = (Jtop.x,Rcb.end.y);
34 TwoTerminal Ic = current((Jbot+Rcb.end)/2, 0, "", "$I_3$");
35 TwoTerminal Rb = resistor(Jtop, normal, -90, "$6\U{\Ohm}$", "");
36 TwoTerminal Ib = current(Rb.end, -90, "", "$I_2$");
37 TwoTerminal Ba = source(Jtop, DC, 180, "", "$6.0\U{V}$");
38 TwoTerminal Ia = current(Ba.end, 180, "$I_1$", "");
39 TwoTerminal Rab = resistor(Jbot, normal, 180, "$8\U{\Ohm}$", "");
40 TwoTerminal Raa = resistor(Rab.end, normal, 90, "$12\U{\Ohm}$", "");
41 wire(Ia.end, Raa.end, rlsq);
42 wire(Jbot, Ib.end, nsq);
43 wire(Jbot, Ic.beg, nsq);
44 wire(Ib.end, Rb.end, nsq);
45 wire(Ic.end, Rcb.end, rlsq);
46 dot("a", Jbot, S);
47 \end{asy}
48 \end{center}
49 Label the resistors from left to right: $R_1 = 12\U{\Ohm}$, $R_2 =
50 8\U{\Ohm}$, $R_3 = 6\U{\Ohm}$, $R_4 = 2\U{\Ohm}$, and $R_5 =
51 10\U{\Ohm}$.
52
53 Label the batteries from left to right: $V_1 = 6.0\U{V}$ and $V_2 =
54 3.0\U{V}$.
55
56 Applying Kirchoff's junction rule to junction $a$ we have
57 $$I_1 + I_2 - I_3 = 0$$
58
59 Applying Kirchoff's loop rule to the left-hand loop we have
60 $$V_1 - I_1 (R_1 + R_2) + R_3 I_2 = 0$$
61 where we \emph{add} the voltage change over $R_3$ because we cross it
62 \emph{against} the direction of the current $I_2$.
63
64 Applying Kirchoff's loop rule to the right-hand loop we have
65 $$V_2 - R_4 I_3 - R_3 I_2 - R_5 I_5 = V_2 - I_3 (R_4 + R_5) - R_3 I_2 = 0$$
66
67 We now have three equations for three unknowns (the $I_i$).
68 Solving the loop rools for $I_1$ and $I_3$ we have
69 \begin{align*}
70   I_1 &= \frac{V_1 + R_3 I_2}{R_1 + R_2} = \frac{V_1 + R_3 I_2}{R_{12}} \\
71   I_3 &= \frac{V_2 - R_3 I_2}{R_4 + R_5} = \frac{V_2 - R_3 I_2}{R_{45}}
72 \end{align*}
73 where we have used the equivalent resistances $R_{12} \equiv R_1 +
74 R_2$ and $R_{45} \equiv R_4 + R_5$ to save writing later.  We can then
75 plug those currents into the junction rule and solve for $I_2$
76 \begin{align*}
77   \frac{V_1 + R_3 I_2}{R_{12}} + I_2 - \frac{V_2 - R_3 I_2}{R_{45}} &= 0 \\
78   \frac{V_1}{R_{12}} + \frac{R_3}{R_{12}} I_2 + I_2 
79     - \frac{V_2}{R_{45}} + \frac{R_3}{R_{45}}I_2 &= 0 \\
80   \p({\frac{R_3}{R_{12}} + 1 + \frac{R_3}{R_{45}}})\cdot I_2
81     &= \frac{V_2}{R_{45}} - \frac{V_1}{R_{12}} \\
82   I_2 &= \frac{\frac{V_2}{R_{45}} - \frac{V_1}{R_{12}}}{\frac{R_3}{R_{12}}
83     + 1 + \frac{R_3}{R_{45}}} \\
84   I_2 &= \ans{-28\U{mA}}
85 \end{align*}
86 Where the $-$ sign means the true current is in the opposite direction
87 to the one we have assigned (so the true current flows upward in the
88 figure).  We can now plug this current in to find $I_1$ and $I_3$.
89 \begin{align*}
90   I_1 &= \frac{V_1 + R_3 I_2}{R_{12}} = \ans{292\U{mA}} \\
91   I_3 &= \frac{V_2 - R_3 I_2}{R_{45}} = \ans{264\U{mA}}
92 \end{align*}
93
94 Double-checking our algebra, we see
95  $I_1 + I_2 - I_3 = 292 - 27 - 264 = -1\U{mA} \approx 0$
96 where difference of $1\U{mA}$ is due to rounding errors from forcing
97 our answers to milli-Volt precision.
98 \end{solution}